[問題]3

[問題]3

ming 於 星期一 五月 05, 2003 5:30 pm


an m*n*p rectangular box has half the volume of an (m+2)(n+2)(p+2) rectangular box where m,n,p are integers and m<=n<=p.what 's the largest value of p?

ming
訪客
 

Raceleader 於 星期一 五月 05, 2003 6:30 pm


左鍵: 點擊縮放; 右鍵: 觀看原圖

Raceleader
訪客
 




代數學